22
$\begingroup$

I was trying to understand completely the post of Terrence Tao on Ax-Grothendieck theorem. This is very cute. Using finite fields you prove that every injective polynomial map $\mathbb C^n\to \mathbb C^n$ is bijective. It seems to me that the only point in the proof presented in the post that is not explained completely is the following lemma:

Take any finitely generated ring over $\mathbb Z$ and quotient it by a maximal ideal. Then the quotient is a finite field.

Is there some comprehensible reference for the proof of this lemma?

In slightly different wording, the question is the following: assuming Nullstellensatz, can one really give a complete proof of Ax-Grothendick theorem in two pages, so that it can be completely explained in one (2 hours) lecture of an undergraduate course on algebraic geometry?

$\endgroup$
4
  • 4
    $\begingroup$ This was previously answered: mathoverflow.net/questions/30599/… $\endgroup$
    – fherzig
    Mar 14, 2011 at 22:19
  • $\begingroup$ I still wonder where this little Lemma appears in the literature ... $\endgroup$ Oct 13, 2013 at 8:52
  • 2
    $\begingroup$ According to Serre, there's a proof of this lemma in Bourbaki, N. Algèbre Commutative. Chapitre V. Entiers, Hermann, Paris, 1964. (p. 68, cor. 1) $\endgroup$ May 19, 2018 at 6:39
  • 2
    $\begingroup$ @MikePierce Indeed I find it p64 (chap 5, §3, no.4). Statement is (translated and summarized) "Every finitely generated [commutative] algebra over $\mathbf{Z}$ is a Jacobson ring; a prime ideal $P$ is maximal iff the quotient ring is finite." $\endgroup$
    – YCor
    May 19, 2018 at 8:50

6 Answers 6

26
$\begingroup$

$\DeclareMathOperator\Spec{Spec}$To prove Nullstellensatz over $\mathbb{Z}$: as the morphism $f: \Spec(R)\to\Spec(\mathbb Z)$ is of finite type, a theorem of Chevalley says that the image of any constructible subset is constructible. So the image of any closed point by $f$ is a point which is a constructible subset. This can not be the generic point of $\Spec(\mathbb Z)$, so it must be a closed point.

Note that this does not hold in general. For example, over the ring of $p$-adic integers, the ideal $(pX-1)\mathbb{Z}_p[X]$ is maximal, but its preimage in $\mathbb{Z}_p$ is $0$ and it not maximal.

[EDIT] Another proof using Noether's normalization lemma: Noether's normalization lemma over a ring A: if a maximal ideal $\mathfrak m$ of $R$ is such that $\mathfrak m\cap \mathbb Z=0$, then $R/\mathfrak m$ is finite type over (and contains) $\mathbb Z$. So there exists $f\in\mathbb Z$ non-zero and a finite injective homomorphism $\mathbb Z_f[X_1,\dotsc, X_d]\hookrightarrow R/\mathfrak m$. But then $\mathbb Z_f[X_1,\dotsc, X_d]$ must be a field. This is impossible because the units of this ring are $\pm f^k$, $k$ relative integers.

$\endgroup$
6
  • $\begingroup$ This first argument is quite nice! $\endgroup$ Mar 6, 2011 at 2:49
  • 2
    $\begingroup$ The theorem of Chevalley can be proved by induction, using the second argument :). $\endgroup$
    – Qing Liu
    Mar 6, 2011 at 10:17
  • $\begingroup$ @Qing Liu, thanks for the answer. I am lost when you say in lines 2-3 of Edit: "So there exits $f\in \mathbb Z$ non-zero and a finite injective homomorphism ...". Why is there such $f$? $\endgroup$
    – aglearner
    Mar 7, 2011 at 0:16
  • $\begingroup$ Sorry, my punctions were not good. I should write "If a maximal ideal ..., so there exists f". The existence of such a $f$ is a form of Noether's normalization lemma that you can find at mathoverflow.net/questions/42276 $\endgroup$
    – Qing Liu
    Mar 7, 2011 at 8:14
  • 1
    $\begingroup$ What does "relative" mean in "$k$ relative integers"? \\ @aglearner, it's not on the web and I don't know if it counts as pedagogical, but Borel discusses Chevalley's theorem in Corollary AG 10.2 of Linear algebraic groups. $\endgroup$
    – LSpice
    Apr 9, 2023 at 20:47
19
$\begingroup$

Let $R$ be a finitely generated $\mathbb{Z}$-algebra, and $\mathfrak{m}\subset R$ a maximal ideal. We wish to show $R/\mathfrak{m}$ is a finite field.

Let $i: \mathbb{Z}\to R$ be the unique ring map; then $i^{-1}(\mathfrak{m})$ is a maximal ideal in $\mathbb{Z}$ (as $R$ is finitely generated over $\mathbb{Z})$, and thus $\mathbb{Z}/i^{-1}(\mathfrak{m})$ is a finite field $\mathbb{F}_p$ for some prime $p$. As $R$ is finitely generated over $\mathbb{Z}$, $R/\mathfrak{m}$ is finitely generated over $\mathbb{F}_p$. But all finite field extensions of $\mathbb{F}_p$ are still finite, completing the proof.

$\endgroup$
14
  • $\begingroup$ Daniel, thanks for the proof of the lemma! $\endgroup$
    – aglearner
    Mar 6, 2011 at 0:16
  • 21
    $\begingroup$ Perhaps it's worth emphasizing that what makes this argument work is the non-trivial fact (proved in Qing Liu's answer) that if $f:\mathbb{Z}\rightarrow\mathbb{R}$ is of finite type, then the pre-image of a maximal ideal of $R$ under $f$ is non-zero. This is a particular case of the general Nullstellensatz on Jacobson rings (found in, e.g., Eisenbud's book). A proof of the statement about fields finitely generated as rings using the usual Nullstellensatz is outlined in Exercise 6 of the Noetherian rings chapter of Atiyah and Macdonald. $\endgroup$ Mar 6, 2011 at 1:07
  • $\begingroup$ @Keenan Kidwell: Of course; thanks for making that explicit. $\endgroup$ Mar 6, 2011 at 1:13
  • 2
    $\begingroup$ An ignorant question: "$R/\mathfrak{m}$ is finitely generated over $\mathbb F_p$. But all finite field extensions of $\mathbb F_p$ are still finite, completing the proof". I'm missing something here. How did finitely generated extension get to be the same as finite field extension? (in particular isn't $\mathbb F_p(x)$ a finitely generated extension but not a finite extension)? $\endgroup$ Mar 6, 2011 at 21:44
  • 1
    $\begingroup$ $\mathbb{F}_p(x)$ is not finitely generated as an $\mathbb{F}_p$-algebra; in particular, all the irreducibles need to be inverted. See e.g. mathreference.com/ag,fgaf.html $\endgroup$ Mar 6, 2011 at 21:54
8
$\begingroup$

Let me add a self-contained answer which is completely elementary and avoids both the Nullstellensatz and Noether's normalization. It is a polished version of Algebrac geometry, finitely generated algebra as a field (already linked in Daniel Litt's comments). It partially overlaps with the answers of Daniel Litt and Guillermo Mantilla.


Lemma 1. Let $R$ be a UFD with infinitely many primes. Then an algebraic field extension $L$ of $F:=\operatorname{Frac}(R)$ cannot be finitely generated as an $R$-algebra.

Proof. Assume $L=R[y_1,\dots,y_n]$, with each $y_j$ being a root of a certain monic polynomial $p_j\in F[x]$. Taking the common denominator $d\in R$ of the coefficients of these polynomials $p_j$, we get that $y_j$ is integral over $R':=R[1/d]\subseteq F$. But then, given a prime $p\nmid d$ (here we use that $R$ has infinitely many primes), the same holds for $1/p\in L=R'[y_1,\dots,y_n]$. However, since $R'$ is still a UFD, this implies $1/p\in R'$ (as a UFD is integrally closed), contradiction. $\blacksquare$

Lemma 2. Given a field extension $K\subseteq L$, if $L$ is finitely generated as a $K$-algebra then the extension is algebraic, and in particular finite (meaning $\operatorname{dim}_K L<\infty$).

Proof. Assume $L=K[z_1,\dots,z_m]$. Since $L=K(z_1)[z_2,\dots,z_m]$, by induction $L$ is algebraic over $K(z_1)$. If $z_1$ is transcendental over $K$, then $R:=K[z_1]\cong K[x]$ satisfies the hypotheses of Lemma 1, which contradicts that $L$ is finitely generated as an $R$-algebra. So $z_1$ is algebraic over $K$, hence also $L$. (Note that the base case $m=1$ is obvious, as $1/z_1\in K[z_1]$ implies that $z_1$ is the root of some polynomial over $K$.) $\blacksquare$

Theorem. If $L$ is a field which is finitely generated, then $L$ is a finite field.

Proof. $L$ is (isomorphic to) the quotient of the ring $\mathbb{Z}[x_1,\dots,x_n]$ by a maximal ideal $M$. Observe that $M\cap\mathbb Z$ is a prime ideal of $\mathbb Z$. If $M\cap\mathbb Z=\{0\}$, then $\mathbb Z$ embeds into $L$; but this contradicts Lemma 1 (with $R:=\mathbb Z$)! Hence $\mathbb F_p$ embeds into $L$ for some prime number $p$ and Lemma 2 gives that $L$ is finite. $\blacksquare$

$\endgroup$
7
$\begingroup$

$\DeclareMathOperator\Max{Max}$One can give a more elementary proof of the fact that $\mathfrak{m} \cap \mathbb{Z} \neq 0$ — By more elementary I mean a proof that only uses the Nullstellensatz over $\mathbb{Q}$.

Notice that it is enough to verify the claim for $R=\mathbb{Z}[x_1,\dotsc,x_n]$, and $\mathfrak{m} \in \Max(R)$.

Suppose there is $\mathfrak{m} \in \Max(R)$ such that $\mathfrak{m} \cap \mathbb{Z} =0$. Then, we may assume that $\mathbb{Z} \subseteq F :=\mathbb{Z}[x_1,\dotsc,x_n]/\mathfrak{m}$. If we denote by $\alpha_{i}=x_i+\mathfrak{m}$ we have that $F=\mathbb{Z}[\alpha_1,\dotsc,\alpha_n]$. Since $F$ is a field we conclude that $\mathbb{Z}[\alpha_1,\dotsc,\alpha_n]=\mathbb{Q}(\alpha_1,\dotsc,\alpha_n)$.

Claim: $F/\mathbb{Q}$ is an algebraic extension.

proof: $F/\mathbb{Q}$ is a finitely generated field extension— generated as an algebra— in particular $F$ is of the form $\mathbb{Q}[y_1,\dotsc,y_m]/M$ for some maximal ideal $M$ of $\mathbb{Q}[y_1,\dotsc,y_m].$ By the Nullstellensatz $M$ has a zero $(\beta_1,\dotsc,\beta_m)$ where each $\beta_i$ is algebraic over $\mathbb{Q}$, so $F=\mathbb{Q}(\beta_1,\dotsc,\beta_m)$ is algebraic over $\mathbb{Q}$.

Since each $\alpha_{i}$ is algebraic, there are integers $q_i$'s such that $q_{i}\alpha_{i}$ is integral over $\mathbb{Z}$ for all $i$. In particular $F=\mathbb{Z}[\alpha_1,\dotsc,\alpha_n]$ is an integral extension of $\displaystyle \mathbb{Z}[\frac{1}{q_1},\dotsc,\frac{1}{q_n}]$. Since $F$ is a field we have that $\displaystyle \mathbb{Z}[\frac{1}{q_1},\dotsc,\frac{1}{q_n}]$ is a field, which is a contradiction ($p$ is not invertible for any prime not dividing $q_{1}\dotsm q_{n}$).

$\endgroup$
2
  • $\begingroup$ Unfortunately, I can not understand when you write " By the Nullstellensatz we have that each $\alpha_i$ is algebraic over $\mathbb Q$". Could you please explain this point? Are you using Nullstelensatz over $\mathbb Q$ here? To which ring are you applying it? $\endgroup$
    – aglearner
    Mar 7, 2011 at 0:12
  • $\begingroup$ @aglearner: I've added an explanation to what you are wondering. The point is that one version of the Nullstellensatz, which I learned by the name algebraic Nullstellensatz, is the following: A finitely generated extensions of fields $F/K$ is algebraic. $\endgroup$ Mar 7, 2011 at 1:21
5
$\begingroup$

This is not an answer to your question, but let me point out that the Ax-Grothendieck theorem is now easy to prove using E-polynomials (Hodge-Deligne polynomials). If $f:X \to X $ is an injective endomorphism of a complex algebraic variety, then $E(X) = E(f(X))=E(X)-E(X\setminus f(X))$. So $E(X\setminus f(X))=0$ and $X\setminus f(X) = \emptyset$, because the degree of a constructible set is twice its dimension. Since one supposes the mixed Hodge theory, this proof is not trivial at all. But at least for me, this looks more natural.

$\endgroup$
3
  • $\begingroup$ I hadn't heard about this, and I don't know anything about E-polynomials. Do you know if the proof you describe uses anything in characteristic p in the background? Any "spreading" over Z? $\endgroup$
    – Marty
    Mar 6, 2011 at 3:17
  • $\begingroup$ This proof uses only the mixed Hodge theory, and one does not need to switch the base field or ring. On the other hand, both proofs by E-polynomial and finite fields have the "motivic" nature. Namely the E-polynomial and the number of rational points are generalized Euler charqcteristics, that is, they have the additivity and multipicativity. $\endgroup$ Mar 6, 2011 at 9:39
  • $\begingroup$ But I wonder, since much of mixed Hodge theory (after Deligne) lifts characteristic p results to get characteristic zero results. And perhaps even transcendental proofs in Hodge theory (like those of Saito) might hide some characteristic p aspects. Do you know if characteristic p is hidden in the background for the results on the E-polynomial? $\endgroup$
    – Marty
    Mar 10, 2011 at 22:05
1
$\begingroup$

This is a direct consequence of Noether Normalization Lemma over the integers:

[Corrigendum to “Noether Normalization theorem and dynamical Gröbner bases over Bezout domains of Krull dimension 1” [J. Algebra 492 (15) (2017) 52-56], Maroua Gamanda, Ihsen Yengui]. See: https://www.sciencedirect.com/science/article/pii/S002186931930314X

It corresponds to the case where one has a positive characteristic $p$ and a finitely generated $\mathbb{F}_p$-vector space.

$\endgroup$
1
  • $\begingroup$ I can't remember if ScienceDirect has done it, but lots of publishers like to shuffle around their links over time, to keep readers on their toes. I find DOIs more reliable: 10.1016/j.jalgebra.2019.06.002. $\endgroup$
    – LSpice
    Apr 9, 2023 at 20:38

Your Answer

By clicking “Post Your Answer”, you agree to our terms of service and acknowledge you have read our privacy policy.

Not the answer you're looking for? Browse other questions tagged or ask your own question.